Combinatorial proof $sum_{i=k}^n {i-1 choose k-1} = {n choose k}$ [duplicate]












0












$begingroup$



This question already has an answer here:




  • Prove ${n - 1 choose k - 1} + {n - 2 choose k - 1} + {n - 3 choose k - 1} + dots + {k - 1 choose k - 1} = {n choose k}$

    3 answers



  • Combinatorial Analysis: Fermat's Combinatorial Identity

    1 answer




Could someone help me as I am stuck with coming up with a proof for this?



Assume n is the total number of people in a town. Assume k is the number of possible ways to select a chief of the town. So the RHS is saying that there are k ways to choose a chief from n people.



on the LHS, From $i=k$, and $k=n$, it is referring to from k to n, which is the sum of the remaining people in the town who were not selected $(n-k)$, that there is $k-1$ ways to choose from $i-1$ objects. Since $i=k$, i could be the number of ways to possibly select a chief. If one person is chosen from i, who also belongs to $k, k-1$. But how does this lead to $${n choose k}$$?










share|cite|improve this question











$endgroup$



marked as duplicate by Sil, N. F. Taussig combinatorics
Users with the  combinatorics badge can single-handedly close combinatorics questions as duplicates and reopen them as needed.

StackExchange.ready(function() {
if (StackExchange.options.isMobile) return;

$('.dupe-hammer-message-hover:not(.hover-bound)').each(function() {
var $hover = $(this).addClass('hover-bound'),
$msg = $hover.siblings('.dupe-hammer-message');

$hover.hover(
function() {
$hover.showInfoMessage('', {
messageElement: $msg.clone().show(),
transient: false,
position: { my: 'bottom left', at: 'top center', offsetTop: -7 },
dismissable: false,
relativeToBody: true
});
},
function() {
StackExchange.helpers.removeMessages();
}
);
});
});
Dec 16 '18 at 16:03


This question has been asked before and already has an answer. If those answers do not fully address your question, please ask a new question.














  • 1




    $begingroup$
    Possible duplicate of Prove ${n - 1 choose k - 1} + {n - 2 choose k - 1} + {n - 3 choose k - 1} + dots + {k - 1 choose k - 1} = {n choose k}$, and also Combinatorial Analysis: Fermat's Combinatorial Identity
    $endgroup$
    – Sil
    Dec 16 '18 at 15:28












  • $begingroup$
    The linked question may help you to obtain an answer, but I think an important prerequisite is to learn how to interpret combinatorial expressions properly. In the quantity $binom{n}{k}$, the number $k$ is the number being chosen, not the number of ways to choose. The very first thing you need to get straight is that $binom{n}{k}$ is the number of ways to choose $k$ items out of a set of $n$ items. In the problem of choosing a single chief from a town of $n$ people, the answer would be $binom{n}{1}$ or $n$.
    $endgroup$
    – Will Orrick
    Dec 16 '18 at 16:18
















0












$begingroup$



This question already has an answer here:




  • Prove ${n - 1 choose k - 1} + {n - 2 choose k - 1} + {n - 3 choose k - 1} + dots + {k - 1 choose k - 1} = {n choose k}$

    3 answers



  • Combinatorial Analysis: Fermat's Combinatorial Identity

    1 answer




Could someone help me as I am stuck with coming up with a proof for this?



Assume n is the total number of people in a town. Assume k is the number of possible ways to select a chief of the town. So the RHS is saying that there are k ways to choose a chief from n people.



on the LHS, From $i=k$, and $k=n$, it is referring to from k to n, which is the sum of the remaining people in the town who were not selected $(n-k)$, that there is $k-1$ ways to choose from $i-1$ objects. Since $i=k$, i could be the number of ways to possibly select a chief. If one person is chosen from i, who also belongs to $k, k-1$. But how does this lead to $${n choose k}$$?










share|cite|improve this question











$endgroup$



marked as duplicate by Sil, N. F. Taussig combinatorics
Users with the  combinatorics badge can single-handedly close combinatorics questions as duplicates and reopen them as needed.

StackExchange.ready(function() {
if (StackExchange.options.isMobile) return;

$('.dupe-hammer-message-hover:not(.hover-bound)').each(function() {
var $hover = $(this).addClass('hover-bound'),
$msg = $hover.siblings('.dupe-hammer-message');

$hover.hover(
function() {
$hover.showInfoMessage('', {
messageElement: $msg.clone().show(),
transient: false,
position: { my: 'bottom left', at: 'top center', offsetTop: -7 },
dismissable: false,
relativeToBody: true
});
},
function() {
StackExchange.helpers.removeMessages();
}
);
});
});
Dec 16 '18 at 16:03


This question has been asked before and already has an answer. If those answers do not fully address your question, please ask a new question.














  • 1




    $begingroup$
    Possible duplicate of Prove ${n - 1 choose k - 1} + {n - 2 choose k - 1} + {n - 3 choose k - 1} + dots + {k - 1 choose k - 1} = {n choose k}$, and also Combinatorial Analysis: Fermat's Combinatorial Identity
    $endgroup$
    – Sil
    Dec 16 '18 at 15:28












  • $begingroup$
    The linked question may help you to obtain an answer, but I think an important prerequisite is to learn how to interpret combinatorial expressions properly. In the quantity $binom{n}{k}$, the number $k$ is the number being chosen, not the number of ways to choose. The very first thing you need to get straight is that $binom{n}{k}$ is the number of ways to choose $k$ items out of a set of $n$ items. In the problem of choosing a single chief from a town of $n$ people, the answer would be $binom{n}{1}$ or $n$.
    $endgroup$
    – Will Orrick
    Dec 16 '18 at 16:18














0












0








0





$begingroup$



This question already has an answer here:




  • Prove ${n - 1 choose k - 1} + {n - 2 choose k - 1} + {n - 3 choose k - 1} + dots + {k - 1 choose k - 1} = {n choose k}$

    3 answers



  • Combinatorial Analysis: Fermat's Combinatorial Identity

    1 answer




Could someone help me as I am stuck with coming up with a proof for this?



Assume n is the total number of people in a town. Assume k is the number of possible ways to select a chief of the town. So the RHS is saying that there are k ways to choose a chief from n people.



on the LHS, From $i=k$, and $k=n$, it is referring to from k to n, which is the sum of the remaining people in the town who were not selected $(n-k)$, that there is $k-1$ ways to choose from $i-1$ objects. Since $i=k$, i could be the number of ways to possibly select a chief. If one person is chosen from i, who also belongs to $k, k-1$. But how does this lead to $${n choose k}$$?










share|cite|improve this question











$endgroup$





This question already has an answer here:




  • Prove ${n - 1 choose k - 1} + {n - 2 choose k - 1} + {n - 3 choose k - 1} + dots + {k - 1 choose k - 1} = {n choose k}$

    3 answers



  • Combinatorial Analysis: Fermat's Combinatorial Identity

    1 answer




Could someone help me as I am stuck with coming up with a proof for this?



Assume n is the total number of people in a town. Assume k is the number of possible ways to select a chief of the town. So the RHS is saying that there are k ways to choose a chief from n people.



on the LHS, From $i=k$, and $k=n$, it is referring to from k to n, which is the sum of the remaining people in the town who were not selected $(n-k)$, that there is $k-1$ ways to choose from $i-1$ objects. Since $i=k$, i could be the number of ways to possibly select a chief. If one person is chosen from i, who also belongs to $k, k-1$. But how does this lead to $${n choose k}$$?





This question already has an answer here:




  • Prove ${n - 1 choose k - 1} + {n - 2 choose k - 1} + {n - 3 choose k - 1} + dots + {k - 1 choose k - 1} = {n choose k}$

    3 answers



  • Combinatorial Analysis: Fermat's Combinatorial Identity

    1 answer








combinatorics discrete-mathematics permutations combinations






share|cite|improve this question















share|cite|improve this question













share|cite|improve this question




share|cite|improve this question








edited Dec 16 '18 at 15:25









Sik Feng Cheong

1579




1579










asked Dec 16 '18 at 15:21









MunchiesOatsMunchiesOats

323




323




marked as duplicate by Sil, N. F. Taussig combinatorics
Users with the  combinatorics badge can single-handedly close combinatorics questions as duplicates and reopen them as needed.

StackExchange.ready(function() {
if (StackExchange.options.isMobile) return;

$('.dupe-hammer-message-hover:not(.hover-bound)').each(function() {
var $hover = $(this).addClass('hover-bound'),
$msg = $hover.siblings('.dupe-hammer-message');

$hover.hover(
function() {
$hover.showInfoMessage('', {
messageElement: $msg.clone().show(),
transient: false,
position: { my: 'bottom left', at: 'top center', offsetTop: -7 },
dismissable: false,
relativeToBody: true
});
},
function() {
StackExchange.helpers.removeMessages();
}
);
});
});
Dec 16 '18 at 16:03


This question has been asked before and already has an answer. If those answers do not fully address your question, please ask a new question.









marked as duplicate by Sil, N. F. Taussig combinatorics
Users with the  combinatorics badge can single-handedly close combinatorics questions as duplicates and reopen them as needed.

StackExchange.ready(function() {
if (StackExchange.options.isMobile) return;

$('.dupe-hammer-message-hover:not(.hover-bound)').each(function() {
var $hover = $(this).addClass('hover-bound'),
$msg = $hover.siblings('.dupe-hammer-message');

$hover.hover(
function() {
$hover.showInfoMessage('', {
messageElement: $msg.clone().show(),
transient: false,
position: { my: 'bottom left', at: 'top center', offsetTop: -7 },
dismissable: false,
relativeToBody: true
});
},
function() {
StackExchange.helpers.removeMessages();
}
);
});
});
Dec 16 '18 at 16:03


This question has been asked before and already has an answer. If those answers do not fully address your question, please ask a new question.










  • 1




    $begingroup$
    Possible duplicate of Prove ${n - 1 choose k - 1} + {n - 2 choose k - 1} + {n - 3 choose k - 1} + dots + {k - 1 choose k - 1} = {n choose k}$, and also Combinatorial Analysis: Fermat's Combinatorial Identity
    $endgroup$
    – Sil
    Dec 16 '18 at 15:28












  • $begingroup$
    The linked question may help you to obtain an answer, but I think an important prerequisite is to learn how to interpret combinatorial expressions properly. In the quantity $binom{n}{k}$, the number $k$ is the number being chosen, not the number of ways to choose. The very first thing you need to get straight is that $binom{n}{k}$ is the number of ways to choose $k$ items out of a set of $n$ items. In the problem of choosing a single chief from a town of $n$ people, the answer would be $binom{n}{1}$ or $n$.
    $endgroup$
    – Will Orrick
    Dec 16 '18 at 16:18














  • 1




    $begingroup$
    Possible duplicate of Prove ${n - 1 choose k - 1} + {n - 2 choose k - 1} + {n - 3 choose k - 1} + dots + {k - 1 choose k - 1} = {n choose k}$, and also Combinatorial Analysis: Fermat's Combinatorial Identity
    $endgroup$
    – Sil
    Dec 16 '18 at 15:28












  • $begingroup$
    The linked question may help you to obtain an answer, but I think an important prerequisite is to learn how to interpret combinatorial expressions properly. In the quantity $binom{n}{k}$, the number $k$ is the number being chosen, not the number of ways to choose. The very first thing you need to get straight is that $binom{n}{k}$ is the number of ways to choose $k$ items out of a set of $n$ items. In the problem of choosing a single chief from a town of $n$ people, the answer would be $binom{n}{1}$ or $n$.
    $endgroup$
    – Will Orrick
    Dec 16 '18 at 16:18








1




1




$begingroup$
Possible duplicate of Prove ${n - 1 choose k - 1} + {n - 2 choose k - 1} + {n - 3 choose k - 1} + dots + {k - 1 choose k - 1} = {n choose k}$, and also Combinatorial Analysis: Fermat's Combinatorial Identity
$endgroup$
– Sil
Dec 16 '18 at 15:28






$begingroup$
Possible duplicate of Prove ${n - 1 choose k - 1} + {n - 2 choose k - 1} + {n - 3 choose k - 1} + dots + {k - 1 choose k - 1} = {n choose k}$, and also Combinatorial Analysis: Fermat's Combinatorial Identity
$endgroup$
– Sil
Dec 16 '18 at 15:28














$begingroup$
The linked question may help you to obtain an answer, but I think an important prerequisite is to learn how to interpret combinatorial expressions properly. In the quantity $binom{n}{k}$, the number $k$ is the number being chosen, not the number of ways to choose. The very first thing you need to get straight is that $binom{n}{k}$ is the number of ways to choose $k$ items out of a set of $n$ items. In the problem of choosing a single chief from a town of $n$ people, the answer would be $binom{n}{1}$ or $n$.
$endgroup$
– Will Orrick
Dec 16 '18 at 16:18




$begingroup$
The linked question may help you to obtain an answer, but I think an important prerequisite is to learn how to interpret combinatorial expressions properly. In the quantity $binom{n}{k}$, the number $k$ is the number being chosen, not the number of ways to choose. The very first thing you need to get straight is that $binom{n}{k}$ is the number of ways to choose $k$ items out of a set of $n$ items. In the problem of choosing a single chief from a town of $n$ people, the answer would be $binom{n}{1}$ or $n$.
$endgroup$
– Will Orrick
Dec 16 '18 at 16:18










1 Answer
1






active

oldest

votes


















0












$begingroup$

Hint: Partition the set of all $k$-element subsets of $[n] = { 1, 2, dots, n }$ according to their greatest elements.



If you must use a 'real-world' example, consider putting the $n$ townfolk in order—say, by height—and counting the number of ways to choose a committee of $k$ people which will be chaired by the tallest person on the committee. The left-hand side partitions the set of all possible committees according to who will be the committee chair.






share|cite|improve this answer









$endgroup$




















    1 Answer
    1






    active

    oldest

    votes








    1 Answer
    1






    active

    oldest

    votes









    active

    oldest

    votes






    active

    oldest

    votes









    0












    $begingroup$

    Hint: Partition the set of all $k$-element subsets of $[n] = { 1, 2, dots, n }$ according to their greatest elements.



    If you must use a 'real-world' example, consider putting the $n$ townfolk in order—say, by height—and counting the number of ways to choose a committee of $k$ people which will be chaired by the tallest person on the committee. The left-hand side partitions the set of all possible committees according to who will be the committee chair.






    share|cite|improve this answer









    $endgroup$


















      0












      $begingroup$

      Hint: Partition the set of all $k$-element subsets of $[n] = { 1, 2, dots, n }$ according to their greatest elements.



      If you must use a 'real-world' example, consider putting the $n$ townfolk in order—say, by height—and counting the number of ways to choose a committee of $k$ people which will be chaired by the tallest person on the committee. The left-hand side partitions the set of all possible committees according to who will be the committee chair.






      share|cite|improve this answer









      $endgroup$
















        0












        0








        0





        $begingroup$

        Hint: Partition the set of all $k$-element subsets of $[n] = { 1, 2, dots, n }$ according to their greatest elements.



        If you must use a 'real-world' example, consider putting the $n$ townfolk in order—say, by height—and counting the number of ways to choose a committee of $k$ people which will be chaired by the tallest person on the committee. The left-hand side partitions the set of all possible committees according to who will be the committee chair.






        share|cite|improve this answer









        $endgroup$



        Hint: Partition the set of all $k$-element subsets of $[n] = { 1, 2, dots, n }$ according to their greatest elements.



        If you must use a 'real-world' example, consider putting the $n$ townfolk in order—say, by height—and counting the number of ways to choose a committee of $k$ people which will be chaired by the tallest person on the committee. The left-hand side partitions the set of all possible committees according to who will be the committee chair.







        share|cite|improve this answer












        share|cite|improve this answer



        share|cite|improve this answer










        answered Dec 16 '18 at 15:27









        Clive NewsteadClive Newstead

        51.9k474136




        51.9k474136















            Popular posts from this blog

            Le Mesnil-Réaume

            Bundesstraße 106

            Ida-Boy-Ed-Garten